LSAT and Law School Admissions Forum

Get expert LSAT preparation and law school admissions advice from PowerScore Test Preparation.

User avatar
 Jonathan Evans
PowerScore Staff
  • PowerScore Staff
  • Posts: 726
  • Joined: Jun 09, 2016
|
#47055
Complete Question Explanation

The correct answer choice is (C).

Image

This game uses a lot of conditional clues; this becomes important when we are trying to determine whether items are selected or not.

The fact that exactly six are selected and exactly two are not means that this is a defined, overloaded grouping-linear game. Once we know which two items are not selected, we may infer that the other six are selected. This is what happens on this problem.

Once we know L is selected for position 6, the first thing we know is that M is not selected. We also know that if R is selected, R will have to be in position 1.

The problem also stipulates that O is in position 2. What does O have an impact on? O affects P and S in the sense that if P is selected, P will be next to both O and S.

The next question we ask ourselves is whether P can be selected. If P were to be selected, then P would have to be next to O and S as mentioned above. This would mean P would have to be in space 3. However, the problem is that this would put S into position 4. This violates a rule. Therefore, P cannot be selected.

Now we know that both M and P are not selected. Thus, we know the other six are.

We can place R in 1 and N in 5. This leaves us with spaces 3 and 4. S cannot be in 4, so S is in 3. V is the only one left for space 4.

S in 3 is the correct answer to this Must Be True Local question.

Get the most out of your LSAT Prep Plus subscription.

Analyze and track your performance with our Testing and Analytics Package.